Friday, January 31, 2014

Guest Post: Rav Yonah Merzbach on Modern Geocentrists

"And the earth abideth forever"-- Is it literal or figurative? (click here for original Hebrew version)

Written by Rav Yonah Merzbach ZT"L, published 1976
Translated by David Ohsie (corrections welcome and encouraged)

Update: Rabbi Dr. Yonah Merzbach (1900-1980) was for many years chief editor of the Encyclopedia Talmudit and former rabbi of Darmstadt, Germany and a faculty member of Yeshivas Kol Torah. [*]

Section 1

Man was commanded to "replenish the earth, and subdue it" (Genesis 1:28) -- the command includes subduing the forces of nature, forces that were hidden in the creation, which was created by the maker of all things.  In order to subdue them, one must investigate and understand them.  It is well known, and a tradition in our hands, that before the end-times there will slowly be revealed to the eyes of all, many of the secrets of the creation, and God's honor will be thereby elevated.

And in truth, within the past few hundred years, and especially in the past few decades, many of the of the secrets of nature have be deciphered by Man.  The vast distances of space in the universe and the paths of the stars have been clarified;  hidden waves invisible to the eye have been revealed.  The structure of the tiny atom has been deciphered, and within it is contained a mighty power capable of destroying worlds, except that its creator restrained it with a mysterious power that holds it together.  If not for that, all things would disintegrate in an instant...Blessed is the one who in his goodness renews each day the act of creation.

And inasmuch as man expands and understands the creation and its secrets, and the image of world is renewed in his eyes, behold he trembles, is fearful and is terrified by the greatness of his Creator and His wisdom, in matters both large and small (see Rambam, Mishneh Torah, Yesodei Hatorah Chapter One, that according to the understanding of his time he mentioned only great things); and the righteous will walk on that path, and will understand and see the divitinity of it, and will add to the love and fear of Him, and the transgressors who close their eyes from seeing, will be stumbled by them.

Section 2

To the masters of the holy spirit, and to them alone, are revealed the ways of nature and its laws from the verses of scripture and the words of our sages of blessed memory.  Others are liable to make errors in this.  The Torah was not written to be a book of natural science, and it was not for this purpose that our sages of blessed memory said what they said.  Whoever wants -- and there were some like these -- to prove that there is a boundary to the sky in the east and the west, or that the earth is flat and not spherical, relied on the statements of our sages of blessed memory and erred, because they didn't understand that "the Torah speaks in the language of man" (see Rambam, Yesodei Hatorah, 1:9 and 1:12) and in like manner spoke our sages of blessed memory and similar things can be found in the figurative language of our prayers and our praises.  These words are only an outer vestment for the extremely deep matters, and riddles by which to express secrets (see the Rambam's introduction to the Commentary on Mishneh).  And there are those instances where they spoke according to the knowledge of their era to the people of their generation.  And as much as mankind enlarges his understanding, he changes his language, however the words of God will stand for eternity and his word stands erect in his laws and in their lofty ideas and thoughts hidden in the garb of their words.

Section 3

"Who is a wise man?  One who learns from any man" -- the word "man" was precise, meaning even a gentile.  And one who says to you there is wisdom among the nations -- believe him; יש אמונה בגויים - חס-ושלום! אל תאמין, and do not learn from others faith and a worldview, but rather knowledge and wisdom.  And the Rambam wrote (laws of the sanctification of the moon, end of chapter 17) that in such matters where the reason for each claim is revealed and the truth is known, we don't concern ourselves with the identity of the author, even if a gentile authored it.  And an aspect of the holiness of our Tannaim was demonstrated in the argument -- which outwardly touches upon scientific matters -- that occurred between the sages of Israel and the sages of the nations, in that which they said that "their words appear truer than our words".

Section 4

For what purpose have all these words been said?  In an article published in "Digleinu" in the Tishrei 5736 issue entitled "revealing faces in the Torah", the author poured out brimstone and fire on the heads of those who "paskened" like Copernicus, that the earth rotates on its axis over the course of a day, rather than the sun and stars revolving around it.  With the holy anger of the jealousy of God and his Torah, he saw in this "position" denial and apostasy.  In truth, he himself writes that there were among the our great decisors -- Rav Yaakov Emden and Rav Moshe Sofer -- who apparently did not see in the position of Copernicus any defect in the foundations of our faith, and one actually agreed to it [1], while the other -- in truth, apparently in accordance with the experts of his generation that he asked, because he himself did not feel himself to be an expert in this matter -- was doubtful because there were arguments on either side of the issue; however, he did not write of any defect in this belief.  However, the author of the article was angry and annoyed and judged as evil and found guilty anyone who accepted this "position".

And he set down his foundations and his proofs on the verses from the story of creation -- about which the Ramban in his commentary on the Torah said that it is "a deep mystery that is not well-understood from the scriptures" -- and from the words of our sages of blessed memory an their statements according to the their outward appearance (see above).  And all of this was written in a era [2] where man can go into outer space and to land -- with precise calculations and understanding of their laws of motion -- his man-made devices on the moon and the planets, and all this is beside the other proofs of the rotation of the earth, all of which has been proven true.  But he, the author, in his article wants to claim that all the of the heavenly bodies, the sun, the moon, and all the stars, among them at huge distances that are counted in millions and billions, revolve in twenty-four hours around the sphere of the earth -- that is to say, if so, with such great speed that they are moving millions and billions of times faster than the speed of light [3].  He hides his face from facts, and then finds others guilty of "revealing faces"?

Section 5

It is not because of the ignorance that is contained in this "jealousy of God" that I am angry and mock, and not even because, through this, he calls believers and the pious [4] deniers and heretics, or because it would be permitted to degrade one that calls me an evil one.

Rather it is because I am concerned with three things:  1) The desecration of God's name, that through this he makes us targets of mockery and laughter in the eyes of others who will mock the denial of reality by those who are God-fearing. 2) And of the purity of belief, that through this he mixes up the concepts of belief which the Torah and Chazal have described, with regard to which beliefs are mandatory and which are not. 3) And because of the danger to to the public, that through this he endangers the belief of those young people who will go out at some point and hear from others that which will open their eyes, and they will infer a general principle from this case in which they were taught ignorance, to respond, God forbid, in the same way to everything that they are taught.

It is incumbent upon sages who write in periodicals to be careful in their words so that they not become a stumbling block. And therefore I wrote these words.

Notes

[*] Biographical details from the preface to the reprint of Rav Merzbach's article in Ohr Yisrael, Dr. Jeremy Brown's New Heavens and a New Earth: The Jewish Reception of Copernican Thought, and the wikipedia entry for Yeshivas Kol Torah.
[1] Rav Emden actually ended up opposing Copernicus although some things that he wrote seemed to indicate possible agreement.
[2] Written in 1976.
[3] According to Relativity, this apparently would not actually violate any physical laws.  On can, if one likes, look at the earth as stationary and the universe revolving without violating any physical laws including the velocity of the stars from that viewpoint.  However, in the ordinary usage of the term, it is the earth that rotates (e.g. the rotation causes an equitorial bulge).
See http://blogs.discovermagazine.com/badastronomy/2010/09/14/geocentrism-seriously/
[4] Literally, חרדים, and I originally translated as Charedim.  This left the a possibly faulty impression of who was being referred to (per commenter "Norm").

Update: Here is the full quotation regarding the "faster-than-light" argument:
There are other arguments used [against geocentrism], and they seem like good ones but in fact they don’t work out in real life. For example, the most obvious one is that distant stars are light years away. If they circle us once per day, they must move faster than light, which is impossible! This is true even for Neptune; at its distance it would have to move at just faster than light to make one circle every 24 hours. 
I thought about this, and wound up asking my friend the cosmologist and fellow Hive Overmind blogger Sean Carroll. He confirmed my thinking: relativity says the math has to work out if you change a frame of reference, so if you do the detailed relativistic equations to look at the motion of distant objects, it still works. Things actually can move faster than light relative to the coordinate system, it’s just that things cannot move past each other with a relative speed greater than light. In the weird geocentric frame where the Universe revolves around the Earth, that is self-consistent. 
In other words, the Neptune-moving-too-quickly argument sounds good, but in reality it doesn’t work, and we shouldn’t use it.

106 comments:

  1. For some reason, Chabad is really committed to geocentrism. They've even enlisted Prof. Branover to "prove" it with all sorts of high-level physics talk.

    ReplyDelete
  2. Note 3 is, I believe, incorrect. The theory of relativity applies only to non-accelerated motion - in a straight line. Revolution is accelerated motion - therefore A revolving around B is not the same as the opposite.

    ReplyDelete
  3. Note 3 is, I believe, incorrect. The theory of relativity applies only to non-accelerated motion - in a straight line. Revolution is accelerated motion - therefore A revolving around B is not the same as the opposite.

    See the linked blog post where he quotes Sean Carroll (who actually understands this stuff) as saying that this "faster than light" counter-argument doesn't work.

    My non-expert understanding that special relativity applies to "rectilinear" motion; however general relativity covers all types of motion.

    Suppose you are in an elevator on the ground floor that starts accelerating upward. From the your frame of reference, it appears that you are in a stronger gravity field so you feel "heavier". So you can view your accelerating frame of reference as actually stationary, but in a gravity field. This equivalence between acceleration and gravity somehow helps explains why gravitational mass always equals inertial mass (in other words, why heavy and light things always fall at the same speed in a gravity field).

    ReplyDelete
  4. I have got to admit that I am actually getting slowly convinced by the geocentric argument.

    I mean what is the chance that after Gallileo and Newton a few hundred years later we would actually find out that geocentricism makes sense from a relativistic standpoint. I mean of course other frames of reference are far more useful for practical science but from a biblical perspective maybe geocentricism really is both the literal and intended meaning!

    ReplyDelete
  5. I updated note 3 to include the full explanation from the cited blog post.

    ReplyDelete
  6. The article you linked to confused me. Is geocentrism really potentially valid, with the only objection being that the math is less simple?

    And as I think about it, let me raise one objection: Can't astronauts see that the earth, in fact, revolves around its axis? For that matter, if they "staioned" themselves somewhere in the middle of space, couldn't they see the earth moving away from them?

    ReplyDelete
  7. Why is the word Charedim in Section 5 par. 1 capitalized? He meant the classic definition - no?

    ReplyDelete
  8. So with the update that was made, I ask, are there any solid irrefutable arguments that the earth revolves around the sun as opposed to the other way around?

    ReplyDelete
  9. For some reason, Chabad is really committed to geocentrism. They've even enlisted Prof. Branover to "prove" it with all sorts of high-level physics talk.

    I think that I need do further research on this and perhaps do a separate post. I think that part of the Chabad argument is based on "relativity says it doesn't matter" which seems to generate a lot of confusion. Dr. Brown's book also tries to deal with this, but his explanation is also confusing to me.

    Of course if it is really true that "relativity says it doesn't matter", then putting the earth at the center can't be an essential belief anyhow, since you could just as well say that the Voyager 2 spacecraft is the center of the universe (as described in the blog post that I linked). Nevertheless, some clarity here would be useful.

    ReplyDelete
  10. A few years back I tried to explain frames of reference on another popular MO blog, but the otherwise intelligent posters just could not grok it. If they haven't studied calculus, physics with relatively, and coordinate transformations, it's a lost cause trying to explain it. Even after taking those courses, I only understand the rudiments.

    As for the heliocentric model, it seems intuitive now (after we learn it in school!) to pick the most massive object in your neighborhood (e.g., your local star) as the center of your coordinate system. As I understand it, the equations are thus vastly simplified, and they actually become solvable for the kinds of calculations we need the most. How could anyone formulate correct equations for earth-based observations based upon the rotating universe that's required by the geocentric model? Heliocentrism is the best arbitrary frame for most of our earthly calculation needs.

    Having said that, geocentrism is convenient if all you need to do is loosely refer to things like sunrise, sunset and arguably the "Zodiac."

    I still don't understand how notions about the "fixed" fabric of space, gravity-space relationships and inertial frames of reference fit into selecting a good frame of reference. Maybe someday somebody will explain it to me.

    ReplyDelete
  11. Norm said...
    Why is the word Charedim in Section 5 par. 1 capitalized? He meant the classic definition - no?


    Good point. I word is capitalized because it is a hebrew word and I wan't sure of his exact intention. I'll switch it to "pious".

    david

    ReplyDelete
  12. So with the update that was made, I ask, are there any solid irrefutable arguments that the earth revolves around the sun as opposed to the other way around?

    The update that was made was simply to quote more of the relevant blog post. What it says is that the argument about whether the earth or stars rotate cannot be settled by the notion that the rotating universe would have the stars going faster than the speed of light, which is impossible. Since Rav Merzbach makes this argument, I felt it important to say that it is apparently not a valid argument.

    The better argument for the absolute rotation of the earth is that the earth bulges out at the equator in the same way that if you spin a bucket, the water will start to move up against the edges of the bucket. This is an observation made by Newton.

    The question of whether or not General Relativity invalidates this or not seems to be an open question based on the research that I have done as a non-expert. I think that a discussion of this deserves another blog post.

    ReplyDelete
  13. The article you linked to confused me. Is geocentrism really potentially valid, with the only objection being that the math is less simple?

    And as I think about it, let me raise one objection: Can't astronauts see that the earth, in fact, revolves around its axis? For that matter, if they "staioned" themselves somewhere in the middle of space, couldn't they see the earth moving away from them?


    There is no middle of space to in which to station themselves. Their velocity is just as arbitrary as anyone else's.

    Is the train moving or is the train stationary and the rest of the universe moving? Relativity says that this is arbitrary.

    The question of rotation seems a bit more complicated.

    ReplyDelete
  14. Yoni said...
    I have got to admit that I am actually getting slowly convinced by the geocentric argument.

    I mean what is the chance that after Gallileo and Newton a few hundred years later we would actually find out that geocentricism makes sense from a relativistic standpoint. I mean of course other frames of reference are far more useful for practical science but from a biblical perspective maybe geocentricism really is both the literal and intended meaning!


    For the sake of argument, let's say that this is so. The conclusion of the post still stands. The pesukim and masorah don't invalidate Copernicus, Kepler, Newton, etc. as the authors thought. Therefore, we should not pasken science from the Torah.

    ReplyDelete
  15. ''The better argument for the absolute rotation of the earth is that the earth bulges out at the equator in the same way that if you spin a bucket, the water will start to move up against the edges of the bucket. This is an observation made by Newton. ''

    possible the most educated Geocentic astronomer is Gerardus D. Bouw, Ph.D.

    so could you respond to his argument that the bulge is explained by mach's principle

    http://www.geocentricity.com/geocentricity/whygeo.html

    hat such is not the case has repeatedly been shown in scientific papers since the turn of the twentieth century.References Back in the nineteenth century, Ernst Mach (photo at left) showed that if there were an essential difference between geocentricity and heliocentrism, that then all the rules of geometry would be violated. Ever since then, geocentricity has been referred to by physicists as Mach's Principle.

    These papers show that the geocentric model is entirely compatible with phenomena such as the stationary satellite, the Foucault pendulum, the equatorial bulge, and how the distant stars can be "moving" faster than the speed of light;

    ReplyDelete
  16. Rotation is much more complicated because it involves the notion of the so-called centrifugal force. This involves the current theory of a fixed entity of a "space fabric," (sort of similar to the old "ether") against which all objects rotate/interact to generate the centrifugal force. Whether or not such a space fabric exists, selecting a reference frame where this centrifugal force cancels out of calculations (e.g., in which the largest object in the local area is not rotating or accelerating) is apparently important to solving equations of motion on a large scale such as our solar system and galaxy.

    From the first reference:
    "So we have non-inertial (accelerating) frames of reference which have the "mysterious" extra forces which don't show up in other frames of reference. In fact, for this reason physicists prefer to write basic equations of motion in inertial frames of reference, which are either at rest or moving at a constant velocity and so do not have any extra forces in them. Centrifugal force doesn't show up in that case then, and this is why sometimes centrifugal force doesn't show up at all in your science textbook and you only learn about centripetal force - that way you can stick with a consistent set of equations of motion and sidestep this whole frames of reference issue."

    So, as I understand it, a geocentric frame of reference is just as valid as any other, but it is a particularly bad one for expressing equations of force and motion. It requires taking into account the rotational forces of all other large objects in the universe, or at least those close enough (e.g., the sun and other planets) to affect earth's movement).

    See
    http://staff.washington.edu/aganse/blog/files/centrip.html
    http://en.wikipedia.org/wiki/Centrifugal_force

    ReplyDelete
  17. I don't see how relativistic arguments help the "frum" Geocentrists. If they claim that the Torah supposedly "proves" geocentrism, then it would have to be the ONLY explation for all the celestial mechanics we see, not just another one that relativisitic arguments allow. The whole argument could then be used against them...we could say "true, relativistic arguments allow for a geocentric perspective, but it also allows a heliocentric one, and not only that, but the law of Universal Gravitition is a lot simpler to write and derive in the heliocentric (i.e. Copernican-Galilean-Newtonian) system than one for the geocentric perspective". Thus, the geocentricists have not had their position proved by the relativists.

    ReplyDelete
  18. I hope someone can answer this question for me:
    Modern technology has allowed us to send spacecraft completely outside the solar system. Two of them are Voyager 1 and Voyager 2, launched in 1977. Voyager 2 has even taken a "family photograph" of most of the planets. What I would like to know regarding the views of the geocentrists is this:
    If we have a spacecraft that goes outside of the solar system in a path perpendicular to the plane the sun and the eight inner planets lie in (i.e. the "ecliptic plane), and it goes far enough away, I am certain it would be able to take time-lapse images (photographs) of the planets moving around the sun, and NOT the planets and sun moving around the earth. What would the geocentrists say about this? Do they say we WOULD see the sun and planets moving around the earth, or do they say that the heliocentric motion which I believe is what would be seen, is an optical illusion?

    I am aware that the modern atomic and quantum physics that was first developed more than a century ago is often defined as "physics that defies common sense" whereas the previously existing Newtonian physics did rely on common sense. However, this "anti-common-sense" physics exists on the atomic scale, on the scale of sizes that are around our size and the world we live in, Newtonian physics works quite well. If we extrapolate out to a scale comparable to the size of the visible universe and a time scale comparable to the age of the universe, it may not be surprising if we again leave the realm of common sense. However, on the size scale of the solar system, we are still within the realm of common sense, and common sense physics tells us that the earth moves around the sun and not the other way around.

    Another point-"stellar aberration" (a slight annual circular motion of certain stars at zenith) was discovered by the English astronomer Bradley early in the 18th century and was viewed as the first direct observation that the earth itself is moving. How do the geocentrists explain that away?

    ReplyDelete
  19. Orbital precession and Centripital effects and warps to space-time due to the can indeed be explained by General Relativistic Fixed Earth reference frame. Common sense would appear to go out the window at that point, but the rotating universe would indeed cause the Earth to bulge.

    ReplyDelete
  20. Rabbi Slifkin used to have a pdf on Zoo Torah of a pamphlet defending geocentrism: I think it was called כצאת השמש בגבורתו. If I remember correctly, it didn't have a good explanation for the Coriolis effect, other than admitting that the Earth is rotating on its axis.

    ReplyDelete
  21. "The better argument for the absolute rotation of the earth is that the earth bulges out at the equator in the same way that if you spin a bucket, the water will start to move up against the edges of the bucket."

    The general theory says that any movement can be explained from any reference system. This means that I can view the earth as motionless and still explain the bulges at the equator. Einstein used Mach's principle to explain it.

    According to relativity theory there is no "right" or "wrong" way of viewing celestial motion. Geocentric is not "correct" or "incorrect" and the same for heliocentric. One may be simpler mathematically that the other but not more correct.

    The Torah always views things from our perspective. As Rav Merzbach quotes, the Torah speaks in the language of man.

    ReplyDelete
  22. ''f we have a spacecraft that goes outside of the solar system in a path perpendicular to the plane the sun and the eight inner planets lie in (i.e. the "ecliptic plane), and it goes far enough away, I am certain it would be able to take time-lapse images (photographs) of the planets moving around the sun, and NOT the planets and sun moving around the earth. What would the geocentrists say about this? Do they say we WOULD see the sun and planets moving around the earth, or do they say that the heliocentric motion which I believe is what would be seen, is an optical illusion?
    ''


    I assume they would say since we cannot know absolute motion. the voyager could be moving in a circle the radius of the earth sun distance. so the sun appears to be still and the planets revolving, but the reverse is happening

    ReplyDelete
  23. more on this here

    https://files.nyu.edu/air1/public/GeoCentrism%20&%20EgoCentrism,%20Existentialist%20Despair%20&%20Significance.htm


    3)All the following statements are FALSE (not only from the scientific viewpoint):
    "science teaches that the earth rotates about the sun",
    "science teaches (or the theory of relativity shows) that the sun rotates about the earth"
    "science cannot determine which one is really - in the scientific sense - rotating about which"
    "science has proven that the earth does NOT rotate about the sun"
    "science has proven that the sun does NOT rotate about the earth"

    Even the following statement is false: "it is equally true scientifically to say that the earth rotates about the sun as to say that the sun rotates about the earth". What is true is that THEY ARE IN RELATIVE ROTATION: and THAT'S ALL THERE IS TO be said from the scientific viewpoint.

    ReplyDelete
  24. so mr einstein if I jump 2 feet in the air, it could be that I have not moved and the earth has dropped 2 feet.

    educashun iz nott wot is craked upp to be. althow I neverr want to hi skooll I wud neverr sei such a daft thingg. yu arr stuppidd

    ReplyDelete
  25. If we have a spacecraft that goes outside of the solar system in a path perpendicular to the plane the sun and the eight inner planets lie in (i.e. the "ecliptic plane), and it goes far enough away, I am certain it would be able to take time-lapse images (photographs) of the planets moving around the sun, and NOT the planets and sun moving around the earth. What would the geocentrists say about this? Do they say we WOULD see the sun and planets moving around the earth, or do they say that the heliocentric motion which I believe is what would be seen, is an optical illusion?

    That is because the spacecraft itself is moving relative to the earth. If it was positioned in such a way that it was "geosynchronous" with the earth, then the earth would appear still and the rest of the solar system moving around it.

    However, you are correct that it is impossible to put a spacecraft in geosynchronous orbit with the earth at the distances of Neptune and beyond because that would involve moving faster than the speed of light. See this article http://www.talkorigins.org/origins/postmonth/2011_05.html for details.

    But hypothetically, if you ignore rotation, I believe that you you could have a spacecraft, with sufficient fuel (maybe an impractically large amount), plot a "shadow" course with the earth. In this case the earth would appear to be rotating, but otherwise in a fixed location with the planets having odd motions that go "around" the earth (in a Tychonic sort of way).

    Which doesn't disprove Copernicus, Kepler or Newton.

    ReplyDelete
  26. Rabbi Slifkin used to have a pdf on Zoo Torah of a pamphlet defending geocentrism: I think it was called כצאת השמש בגבורתו. If I remember correctly, it didn't have a good explanation for the Coriolis effect, other than admitting that the Earth is rotating on its axis.

    This?

    http://www.zootorah.com/controversy/mevohashemesh.pdf

    ReplyDelete
  27. Can you put a pdf up of the original article in Hebrew?

    There is already a link there to the original on hebrewbooks.org. I edited the post to make it more obvious where to click.

    Please let me know of any corrections. I don't really speak Hebrew, so there are undoubtedly small errors of nuance in there, at least.

    ReplyDelete
  28. The general theory says that any movement can be explained from any reference system.

    According to this article, that is incorrect. There is no reference frame where the earth is not rotating, while there could be a coordinate system. For a reference frame you need theoretical reference clocks spread through space in your coordinate system, but this is not possible in an "earth stationary" coordinate system because the clocks have to move faster than the speed of light.


    This means that I can view the earth as motionless and still explain the bulges at the equator. Einstein used Mach's principle to explain it.

    Based on what I have read:

    1) Mach's principles were not specific enough to actually predict anything. You can get multiple possible contradictory predictions from his general principle.

    2) His principle may not be true. Google around for things like "is space machian" and you'll see what I mean.

    ReplyDelete
  29. Just thinking some more about this...

    I think it's important to distinguish between rotation around an internal axis access versus revolution around another body. As I understand it, rotation appears to take place against a a kind of fixed space fabric coordinate system that apparently is not subject to the position, or movement of relative coordinate systems. Rotation around an internal axis generates apparent forces, such as the centrifugal/centripetal forces. These forces exist in every coordinate system, although with different force vectors.

    However, in a many body system (2+ bodies), relative coordinate systems come into play. Those coordinate systems may be moving and rotating or revolving around another body. It seems that the rotation of a coordinate system is not the same as a coordinate system that revolves around another body.

    I am way over my head here, but it's possible to imagine a rotating coordinate system centered at earth's center, and rotating in sync with the earth. In such a system, the planetary and galactic objects would appear to move around the earth in bizarre patterns governed by the strange "fictitious forces" needed to make the math work out. However, the entire notion of a rotating coordinate system is itself bizarre because rotation itself may be against a fixed space fabric that somehow generates rotational forces on spinning masses. These forces may be generated by induced warps in the local space-time fabric due to the spinning mass. I believe that this notion of a kind of absolute rotation (unaffected by reference frames) is under debate at the moment. A rotating coordinate system would seem like a very bad idea in a universe with a "fixed grid" against which all objects rotate. The fixed earth model uses such a rotating coordinate system that does not model the putative physical reality of a fixed space-time fabric.

    On a solid rotating mass, all matter rotates with the same angular velocity. However, for a star and planet 2-body system, both objects may be non-rotating, while the planet travels (revolves) around the star, constrained by the star's gravitational force and the planet's angular velocity. This is different than rotation of a single body. It's possible to use a stationary/moving/accelerating/rotating coordinate system to describe this behavior. If we use a stationary coord system, what is the definition of stationary?

    Stationary relative to what? A fixed space fabric? The galaxy's center? The star's center? The galaxy center's rotation and translation? Is rotation fixed, but circulinear/translational motion always relative? Are rotation, translation and circulinear motion all fixed against a space-time grid - or it it only rotation?

    ReplyDelete
  30. Please excuse this poor ignorant soul: From a Scientific perspective, if General Relativity cannot distinguish between a heliocentric and geocentric model of the universe, then surely this is the incorrect model against which to test that hypothesis? The theory is only valid in as much as it can establish testable hypothesis to distinguish itself from other theories.

    Secondly, in the eptistemology of science Occam's Razor has a place. Simply, man has placed satellites in orbit, and even beyond the solar system. To do this they relied on calculations. These calculation, although, from what I am reading hear, could have assumed that earth is at the centre of everything, are simpler to calculate assuming the earth revolves around the sun.

    Finally, the motion of the train: I seem to recall in first year physics the text book talking about the time-dilation effect, and how do we know which individual will get older etc. I though the answer was that only one of the individuals accelerates, thus the time dilation effect works on that individual. Further in the motion of the train, couldn't we apply thermodynamic principles (i.e. the train produces heat to maintain its motion) to determine which one is moving?

    ReplyDelete
  31. I'm getting a bit restless about the use of relativity to question whether or not it is appropriate to speak of the earth rotating on its axis vs. the universe spinning about the earth. It's a question of realism vs. some mathematical formalism whether of the Machean variety or Einsteinian relativity. Let me give a simple, concrete example. A child riding on a merry-go-round sees his parents and everyone else (and structures) spinning. The adults realize, however, that it is the merry-go-round which is spinning while the world is ostensibly stationary. Who is right? Even the adults in this example don't feel the rotation of the earth, yet various lines of evidence point to that phenomenon. Besides what has already been mentioned, there is the fact that the earth's spin appears to be slowing and has slowed over the eons (the 24 hour day is getting longer). This phenomenon is attributed in classical physics to the tidal interaction with the moon which results in angular momentum loss by the earth coupled with a nearly corresponding gain by the moon. Now, a Machean interpretation could be advanced to the effect that the 'universal rotation' is slowing. Which is simpler, more realistic, and calculable?

    ReplyDelete
  32. Please excuse this poor ignorant soul: From a Scientific perspective, if General Relativity cannot distinguish between a heliocentric and geocentric model of the universe, then surely this is the incorrect model against which to test that hypothesis? The theory is only valid in as much as it can establish testable hypothesis to distinguish itself from other theories.

    Yes, but you should draw the opposite conclusion. If, under a correct model of the universe, there is no way to distinguish between A and B, then A and B are not really different, even though you call one thing A and one thing B.

    Finally, the motion of the train: I seem to recall in first year physics the text book talking about the time-dilation effect, and how do we know which individual will get older etc. I though the answer was that only one of the individuals accelerates, thus the time dilation effect works on that individual. Further in the motion of the train, couldn't we apply thermodynamic principles (i.e. the train produces heat to maintain its motion) to determine which one is moving?

    Unfortunately, no. You'd get the same effect by moving the rails under the train and a blowing a wind equal to the speed of the train, but keeping the train stationary relative to the ground.

    ReplyDelete
  33. I think it's important to distinguish between rotation around an internal axis access versus revolution around another body. As I understand it, rotation appears to take place against a a kind of fixed space fabric coordinate system that apparently is not subject to the position, or movement of relative coordinate systems. Rotation around an internal axis generates apparent forces, such as the centrifugal/centripetal forces. These forces exist in every coordinate system, although with different force vectors.

    The frames for rotation and "revolution" are both non-inertial reference frames (there is acceleration involved). Thus if you sit in a space ship suspended from the ground, you'll feel the force of floor on your shoes, but if you stand in an orbiting space ship (revolving around the earth), you won't.

    ReplyDelete

  34. The frames for rotation and "revolution" are both non-inertial reference frames (there is acceleration involved). Thus if you sit in a space ship suspended from the ground, you'll feel the force of floor on your shoes, but if you stand in an orbiting space ship (revolving around the earth), you won't.


    Sorry, but I'm not sure what your point is. Regardless, imagine a spinning (rotating) person in an otherwise empty universe. Regardless of the coordinate system or frame of reference, the person's arm will tend to fly outward. This means that there appears to be a rotational "grid," against which rotation can be measured, via the force outward force that drives the arms apart. The question of whether such a fixed universal reference frame exists (for rotation in this case) is the main point I was trying to make.

    Revolution around an object seems more similar to circular translation (yes, acceleration is present), whereas pure rotation may be a different beast entirely. These points may be related to to the nature of relative motion and relativistic effects, but I don't claim to understand this.

    ReplyDelete
  35. Besides what has already been mentioned, there is the fact that the earth's spin appears to be slowing and has slowed over the eons (the 24 hour day is getting longer). This phenomenon is attributed in classical physics to the tidal interaction with the moon which results in angular momentum loss by the earth coupled with a nearly corresponding gain by the moon. Now, a Machean interpretation could be advanced to the effect that the 'universal rotation' is slowing. Which is simpler, more realistic, and calculable?

    Not really arguing, but these kinds of arguments always seem to end up with "mushy" words at the end like "simpler". Not "simpler" does not equal not "correct". "Harder to calculate" does not equal "not correct". "Realistic" is not precise to begin with (although there is a sense in which I think that we can make it precise for our purposes).

    So there is a lack of precision here. I have some thoughts on making it more precise, at least as far as our question is concerned, but need time to write it out.

    ReplyDelete
  36. Putting it another way, if you are traveling on a train in a straight line at a constant speed, it seems simpler, more realistic, and more easily calculable to consider the train to be moving, rather than earth moving the other direction. But we know that, in that case, it really is completely arbitrary (in the sense that no experiement that you can do in the train can tell you whether or not you are moving or the earth is). So it has to be a bit more precise.

    ReplyDelete
  37. Unfortunately, no. You'd get the same effect by moving the rails under the train and a blowing a wind equal to the speed of the train, but keeping the train stationary relative to the ground.

    Yes you would, but there would still be "extra" energy to explain the phenomena. In your analogy, the energy expenditure of the train offsets "added" energy from the wind.

    If, under a correct model of the universe, there is no way to distinguish between A and B, then A and B are not really different, even though you call one thing A and one thing B.

    So there is no difference between heliocentrism and geocentrism? That is the conclusion that you have inferred.

    ReplyDelete
  38. Sorry, but I'm not sure what your point is.

    That revolving like spinning.

    Regardless, imagine a spinning (rotating) person in an otherwise empty universe. . Regardless of the coordinate system or frame of reference, the person's arm will tend to fly outward. This means that there appears to be a rotational "grid," against which rotation can be measured, via the force outward force that drives the arms apart.

    It may not be so simple. It may actually rotation relative to the "fixed stars" that cause this to happen. It is not clear what happens if you are the only thing rotating in otherwise empty universe. For example, imagine that all the matter in the entire universe started rotating just now. Would we know? Would it mean anything?

    Like Y. Aharon, I don't think that any of this is all that important. Even if Mach is right and even if a coordinate system with the earth unmoving can be worked out, Kepler was right and Ptolemy wrong about the paths of the planets and Rav Eybeshutz was mistaken in his assertion otherwise. I also don't think that the really saves the Chabad position either. But as I mentioned, it would be good to get more clarity.

    ReplyDelete
  39. Based on everything I'm reading here, it would seem that we are wrong to teach children in school that the earth revolves around the sun. Is that really true?
    If yes, I find it staggering.

    ReplyDelete
  40. This presentation of the formalism helped me see the geocentric point of view, so to speak.

    But it's needless to say that the geocentric frame includes many amazing coincidences. Why does the elliptical motion of the Universe correspond so precisely with the orbit of the Sun around the Earth? Why do perturbations in the Universe's rotation correspond with subtle shifts in the Earth's mass (e.g.)?

    My understanding is that the Lubavitcher Rebbe saw this frame as the absolute truth, and that science is simply useless to distinguish it because science is relative. Not so easy to digest, but perhaps no less easy than the claim that God created fossils in the ground.

    ReplyDelete
  41. I don't understand why this issue is so important for Chabad and other people? Do they claim that since the RAMBAM describes the Ptolemaic system in the Mishna Torah, it must be "Torah MiSinai" and that a corollary is that if we admit the RAMBAM is wrong about this, people may conclude that everything their teachers is wrong and they might as well throw out all of Judaism? Is that the real issue?
    Rav Yaakov Kamenetsky watched the first Moon landing and upon seeing the rocks on the Moon, concluded that the RAMBAM got his science from the Greeks. What is so threatening about such a position, especially if a Torah giant like Rav Yaakov didn't have a problem with it?

    ReplyDelete
  42. Based on everything I'm reading here, it would seem that we are wrong to teach children in school that the earth revolves around the sun. Is that really true?
    If yes, I find it staggering.


    In a word, no. Don't change your current prescription unless advised by a physicist.

    But I suppose the opposite view could be useful in some circumstances: "Sorry, Johnny, we have no choice but to go the dentist now. In fact, we're not moving at all; the dentist's office is coming to us!".

    ReplyDelete
  43. Yossi said...
    Unfortunately, no. You'd get the same effect by moving the rails under the train and a blowing a wind equal to the speed of the train, but keeping the train stationary relative to the ground.


    Yes you would, but there would still be "extra" energy to explain the phenomena. In your analogy, the energy expenditure of the train offsets "added" energy from the wind.

    No extra energy. If you are paddling upstream and just keeping up with the current, is there extra energy?

    If, under a correct model of the universe, there is no way to distinguish between A and B, then A and B are not really different, even though you call one thing A and one thing B.

    So there is no difference between heliocentrism and geocentrism? That is the conclusion that you have inferred

    No conclusions, really, because I don't understand the physics well enough and because Geocentrism and Heliocentrism are not precisly defined here.

    But if Geocentrism = Ptolemy and Heliocentrism = Kepler, then no, they are not equivalent because they give different predictions.

    And if Heliocentrism = "A model of of the world" and Geocentrism = "there exists some equivalent earth centered model that would also explain and predict the motions of the cosmos", then again, not exactly, since one is a model and one is a hypothetical model that has not been worked out yet, even if relativity tells us that it could be worked out.

    ReplyDelete
  44. David, I don't think your recent responses are correct. Although one can use any coordinate system (based on any xyz position and linear velocity vector) to describe an objects location and linear velocity, I do not think that's true for rotating objects. That's because there appears to be a universal notion of rotation relative to a fixed space grid. Although you can devise a rotational coordinate system, you would have to introduce fictional forces to make it work properly with the real universal laws (physics). A geocentric earth requires a rotating coordinate system relative to a fixed space grid. That does not appear to match with the physical reality based on the notion of a fixed grid against which all objects rotate.

    Therefore, a geocentric earth can theoretically be made to work with bizarre mathematical descriptions, but *it does not appear to represent the physical reality.* The notion of a fixed space grid and non-relativistic rotation is not my invention. Again, revolution is not necessarily the same thing as rotation in a relativistic sense, since revolution also involves translation (xyz movement) that may be described by relative coordinate systems.

    I don't understand why this apparent physical reality exists, but it's been postulated by some pretty smart people.

    ReplyDelete
  45. No extra energy. If you are paddling upstream and just keeping up with the current, is there extra energy?

    Obviously YES due to the movement of the water due to gravity. That was why you needed to add the phrase "just keeping up with the current".

    not exactly, since one is a model and one is a hypothetical model that has not been worked out yet, even if relativity tells us that it could be worked out.

    Which brings me back to my original points which were:

    1) If the two models proposed do not make predictions which will allow for the models to be tested (falsified or distinguished from each other) then General Relativity has zero value.

    To put it another way, there should be some experiment that can be established that can verify one model (solution) over another. If we cannot distinguish between the two models by observation, then the theory is the wrong theory to draw resolve the discussion.

    If I am not mistaken (and this is an argument from ignorance), one of the key criticism of string theory is that experimentally, no predictions can be made that would distinguish one model from another.

    2) Occam's razor remains an integral part of the epistemology of science! Science has never hidden this, or been ashamed of it.

    ReplyDelete
  46. David, there comes a point when common sense must rule - even granting the success of the relativity theories. Those theories merely posit that there is no preferred reference frame and that the equations in physics must reflect that. What has that to do with a realistic view of the universe? Yossi's point about energy is valid. You could measure the heat energy given off by the locomotive engine and the energy (voltage x current x time) imparted to it. You can also observe and measure the rotational speed of the electric motor involved. A further complication for a Machean approach is the acceleration or slowing of the train. Does anyone believe that the engine isn't running and that the train's motion isn't changing, but that its the earth or space that is undergoing change. If that is your understanding of reality then you have more problems than accomodating a heliocentric solar system into your religious framework.

    Let's face it. The sages were sometimes, if not often, wrong in their understanding of the physical world - as was the Rambam. The latter adopted the Aristotelian view of the universe then current in educated circles (other than the eternity of the universe). He was wrong about circular motion requiring an intelligent controller, or that gravity is based on things returning to their origin. So what! Why should that cause a problem for religious belief. It is totally arbitrary and opposed to a knowledgeable view of the world to assume that somehow they were always right, or the corollary that their pronouncements were always guided by the divine spirit.

    ReplyDelete
  47. Y. Aharon and Yossi:

    You are actually unwittingly making argument for Geocentrism!

    If we leave rotation out of the picture as you do with the non-accelerating train example, then we are dealing with special relativity. In this case, both the theory and our experience tell us that there is *no* experiment in the world that you can do to tell who is moving and who is standing still. The exact same simple laws apply. No odd fictitious forces and no causality paradoxes.

    In fact, this was known to Newton. Newton's laws are invariant to change in inertial (non-accelerating) reference frames. The problem that Einstein faced was that Maxwell's equations implied that light traveled at a constant speed "c". But that would mean that you could tell how fast you were going: if you see that light is travelling at c - v, then you must be going "v". Then it turned out experimentally, that light was going at speed "c" no matter which direction you moved. So Einstein patched things up to put us back to where Newton was: you can't tell how fast you are going.

    Now, one of the main arguments of the Geocentrists was as follows: if the earth were moving around the Sun, then it would have to do so at incredible speeds. If so, we would know that. Cannonballs would travel faster in one direction than another. A ball dropped from a tower should not land at its base, etc. Somehow we would be able to tell if we were moving that fast. Galilean and then special relativity tells us that this intuition is false.

    In addition, Kepler's laws work because of that fact. The Sun, of course, is not stationary; it is moving at incredible speeds around the center of the galaxy (and the galaxy in the universe). Because of relativity, we can ignore all of that and just look at the motions of the planets and the sun.

    This is why precision in the argument is needed. Yes, intuitively, and for practical everyday life, the ground is still and the train is moving. Taking that as a fundamental truth means that the ground is still which implies ... Geocentrism! (a non-moving earth).

    The right argument has to do with the fact that acceleration can be detected. That was Newton's bucket experiment. That explains the bulge of the Earth at the equator. Except that General Relativity has something to say about that, what it says is a bit beyond me, and you will get different answers depending on where you look.

    Of course, I agree with everything you say about the Rambam. And Chabad's argument, even if accepted, doesn't save the Rambam from "error". And R. Ebyeshutz was mistaken, because he thought that Newton was wrong. But if you want to get it down right, you have avoid arguments about moving trains generating heat, because these are incorrect arguments.

    ReplyDelete
  48. David, I don't think your recent responses are correct. Although one can use any coordinate system (based on any xyz position and linear velocity vector) to describe an objects location and linear velocity, I do not think that's true for rotating objects. That's because there appears to be a universal notion of rotation relative to a fixed space grid.

    I don't claim otherwise, but you can also find the claim that these effects can be attributed to the motion of the stellar masses, not to an absolute space. The "fictitious" accelerations are attributable to gravity. I'm not claiming to understand it, but this is what some people who are smarter claim.

    ReplyDelete
  49. Just to avoid arguing this further given my lack of qualifications, I put up a post with a link to two seemingly opposite points of view on the issue from actual Physicists. You can argue with them instead of me :).

    ReplyDelete
  50. Y.Aharon and Yossi:

    You are actually unwittingly making argument for Geocentrism! David Ohsie

    Huh? We were discussing your moving train relative to the earth, not relative to space. The argument is that it is manifestly unreal to propose that the train is still while the earth moves (or, even more, that the rails move). Those who insist on a simple biblical picture of a stationary earth are forced to such contortions as a spinning space to account for the fact that satellites placed in a polar orbit manage to take images of the entire earth. The advances of 20th century physics in formulating the counter intuitive principles of quantum mechanics and relativity doesn't mean that we should now surrender our sense of what is real and accept a new reality based on a literal understanding of some biblical verses.

    ReplyDelete
  51. No extra energy. If you are paddling upstream and just keeping up with the current, is there extra energy?

    Obviously YES due to the movement of the water due to gravity.

    You said that you could tell the train was moving because it was generating heat. The upstream swimmer is generating heat, and is not moving relative to the ground.

    If you want to look at the stream, we can do so. If stream itself is moving at constant speed due to gravity and the friction with the streambed, then it is an inertial frame. You get the same thing if you have the water "still" but rushing down an inclined slope which is turning under it like an escalator. This is special relativity and there are no fictitious forces.

    That was why you needed to add the phrase "just keeping up with the current".

    No, I was just describing a situation where you would agree the swimmer is not moving, but is generating energy. Is the water moving or the swimmer? Depends on your frame; both are equally valid.

    not exactly, since one is a model and one is a hypothetical model that has not been worked out yet, even if relativity tells us that it could be worked out.

    Which brings me back to my original points which were:

    1) If the two models proposed do not make predictions which will allow for the models to be tested (falsified or distinguished from each other) then General Relativity has zero value.


    No, it means that your models are equivalent or two ways of stating the same thing. Just like A is moving in a straight line and A is stationary is really the same thing in two different reference frames.

    To put it another way, there should be some experiment that can be established that can verify one model (solution) over another. If we cannot distinguish between the two models by observation, then the theory is the wrong theory to draw resolve the discussion.

    No, it means that your models are equivalent and predict the same thing.

    If I am not mistaken (and this is an argument from ignorance), one of the key criticism of string theory is that experimentally, no predictions can be made that would distinguish one model from another.

    Relativity has no such problem; it has been validated many times over.

    ReplyDelete
  52. Huh? We were discussing your moving train relative to the earth, not relative to space. The argument is that it is manifestly unreal to propose that the train is still while the earth moves (or, even more, that the rails move).

    Your intuitive logic tell you that the train is moving while the ground is still. Relativity tells you that this is actually arbitrary (and again, no accelerations involved, assuming a very smooth ride, so no paradoxes of any kind). It really is arbitrary. This is not like the rotating frame because it is intertial. A rotating frame accelerates.

    If you use intuition, then you can say that the Earth is not moving. If it was I could tell. It is relativity that tells you that even though you don't feel yourself moving, that there is a frame of reference where you are (actually infinitely many).

    ReplyDelete
  53. David, again I must assert that relativity is a formalism that is based on the equivalence of relative motions (or vertical acceleration vis-a-vis gravity). It insists that a 'correct' mathematical description of such motion would be independent of reference frame. That is not the same as stating that in real life, we can't make any assumption about which is moving the train or the tracks or the earth (or the carousel vs the bystanders). Besides in real trains there is acceleration and deceleration that can't be so off-handedly transmuted to a corresponding opposite acceleration and deceleration of the earth or space. It seems that you're making modern physics into an irrational endeavor.

    ReplyDelete
  54. David, again I must assert that relativity is a formalism that is based on the equivalence of relative motions (or vertical acceleration vis-a-vis gravity). It insists that a 'correct' mathematical description of such motion would be independent of reference frame. That is not the same as stating that in real life, we can't make any assumption about which is moving the train or the tracks

    This first part here is wrong. It is not just a formalism. There is no experiment using any apparatus or intelligence or anything that you can do to say who is moving and who is not when there is no acceleration. The heat of the engine does not tell you; size of the objects don't tell you. This is physics, not mathematics. This is precisely why we don't sense the fact that we are moving at incredible speeds around the sun and the center of the galaxy and away from other stars and precisely why many of the old proofs for geocentrism were wrong (the other proofs were wrong in other ways).

    or the earth (or the carousel vs the bystanders). Besides in real trains there is acceleration and deceleration that can't be so off-handedly transmuted to a corresponding opposite acceleration and deceleration of the earth or space.

    This part is correct. When you get to accelerations then you get these fictitious forces. That is where you get to the part of relativity that I don't understand, and thus rely on quoting others. There seems to be different ways of looking at that part.

    It seems that you're making modern physics into an irrational endeavor.

    This is a mushy argument. Is the double slit experiment irrational? Is the fact when A is moving and B is not, both A and B see light moving past them at speed "c" irrational?

    ReplyDelete
  55. You said that you could tell the train was moving because it was generating heat. The upstream swimmer is generating heat, and is not moving relative to the ground.

    If you want to look at the stream, we can do so. If stream itself is moving at constant speed due to gravity and the friction with the streambed, then it is an inertial frame. You get the same thing if you have the water "still" but rushing down an inclined slope which is turning under it like an escalator. This is special relativity and there are no fictitious forces.


    Hi David,

    I wasn't foing to continue this discussion in deference to your 7:48 pm post, however since you commented afterwards, I take this as permission to continue.

    Regarding the quote above; Sorry, even my first year university physics from 20 years age can tell you that what you wrote was wrong.

    While the vector analysis of the direction of the energy = zero in the scenario, the total energy expenditure (the kinetic energy) is the sum of the energy in the motion of water AND the energy spent by the swimmer moving!

    And I do not know which fictitious forces forces I have used in this argument (gravity?).

    I am going out on a limb here, but I think that you are negating thermodynamics as you are arguing relativity.

    Yossi

    ReplyDelete
  56. David,

    I believe YA is actually a physicist, so you ought to take his explanations more seriously.

    I think that you are conflating relative reference frames with relative forces and relative causation. Yes, you can devise many reference frames that mangle the notion of causation, force vectors, rotation, and even perhaps the direction of time. However, those frames are not necessarily correct descriptions of the physical reality in our real world; they only represent theoretical "worlds views" that we can represent mathematically.

    In the real world, the evidence shows that forces seem to have fixed causes and effects, and these forces and effects generally have a preferred reference frame for describing them mathematically. Sure, you can use another relative reference frame, but that does not make it a good model of the real world, given our understanding of basic physical laws.

    Your choice of reference frame depends on what you are trying to model. If you only want to model sunrise and sunset, and ignore all else, then a geocentric frame is somewhat reasonable, as a first approximation. Once you add in any other celestial body, it is a terrible reference frame because it ignores the cause and effect of real forces that act on celestial bodies.

    At least that's my take on it.

    ReplyDelete
  57. Your intuitive logic tell you that the train is moving while the ground is still. Relativity tells you that this is actually arbitrary (and again, no accelerations involved, assuming a very smooth ride, so no paradoxes of any kind). It really is arbitrary.

    Only arbitrary in General Relativity, not in thermodynamics. The motion of the train (particularly as it is the object that must accelerate) is measurable, and visible. The observer can actually see the mechanics of the trains motions.

    ReplyDelete
  58. This is frustrating. David apparently believes that if the locomotive engine is operating and the train wheels are turning in tandem with the motor, this is insufficient evidence that the train is moving rather than the background. I guess the wheels are just spinning in place due to a lack of friction with the tracks. The work and heat energy output of the engine also doesn't appear to count. The mantra about not being able to distinguish absolute motion appears to override common sense. Nor is this example one of those strange quantum phenomena or near-the-speed-of-light 'peculiarities'. It should follow 'ordinary' physics and common sense.

    ReplyDelete
  59. This is frustrating. David apparently believes that if the locomotive engine is operating and the train wheels are turning in tandem with the motor, this is insufficient evidence that the train is moving rather than the background.

    1) Thought experiment. I imagine that the train is a toy train and that the tracks are sitting on a table sitting in an ice rink.

    Experiment A: Turn on the to train. You get the wheels turning heat generated, train wheels moving in tandem with the motor, etc. and the train moving at speed S.

    Experiment B: First start the table sliding at speed S. You then turn on the train at speed S going the opposite direction and see the wheels turning, heat generated, etc, but the train is "not moving".

    2) It is frustrating, because the special relativity tells us that the laws are exactly the same in all frames moving at constant velocity relative to each other. This is empirically verified. If you could come up with any experiment that could detect the difference between the two frames, then you would be overturning the basics of physics and receive a Nobel prize. If your intuition goes against this, then your intuition is just wrong. There is no exception for heat, engines, wheels turning or anything else.

    3) This is one of the key reasons that we didn't notice until recently that we are moving really fast relative to the "fixed stars".

    ReplyDelete
  60. Your intuitive logic tell you that the train is moving while the ground is still. Relativity tells you that this is actually arbitrary (and again, no accelerations involved, assuming a very smooth ride, so no paradoxes of any kind). It really is arbitrary.

    Only arbitrary in General Relativity, not in thermodynamics. The motion of the train (particularly as it is the object that must accelerate) is measurable, and visible. The observer can actually see the mechanics of the trains motions

    Again, eliminate the acceleration. That is not an inertial frame. While the train is moving at constant speed you can see still see the mechanics of the train motions. That all fits in special relativity. And there simply no exception for thermodynamics. As long as we are dealing with inertial frames, there is no way to tell which one is at rest and which one is "moving". The laws are exactly the same for both.

    ReplyDelete
  61. And there simply no exception for thermodynamics.

    Yes there is! Conservation of energy demands that the sum energy lost from the system = 0. Find where the "excess" heat is and you will find which object is converting energy from one source to another.

    In the train analogy, the excess heat comes from (1) the motor and (2) friction at different locations.

    The motor is not getting hot to push the earth around! It drives the wheels, converting more energy (via friction) into motion along the tracks.

    ReplyDelete
  62. David,

    I believe YA is actually a physicist, so you ought to take his explanations more seriously.


    I believe he is a chemist and a scientist while I'm just a computer guy. But that doesn't change what special relativity tells us. And it tells us that as long as the train/earth/sun is not accelerating, the laws are the same no matter what frame you are in.

    The rest of what you get into has to do with non-inertial frames. Those, I don't understand other than to say that General Relativity has something to say about it.

    So if your argument is: but we know the train is moving (just look at that engine), then you are wrong and copying an argument from the Geocentrists that Newton had already answered at low speeds compared to the speed of light with Galilean relativity.

    If your argument is: but the earth bulges at the equator, so it seems simple that we have absolute rotation, you are where Newton was, but then General Relativity says something about that and I honestly don't understand what it says exactly.

    ReplyDelete
  63. And there simply no exception for thermodynamics.

    Yes there is!

    I don't know what to say other than "no, there isn't". If you think this is so, please cite something. There are no exceptions to Special Relativity and if you find one, then you are either making a mistake or deserve a Nobel Prize.

    Conservation of energy demands that the sum energy lost from the system = 0. Find where the "excess" heat is and you will find which object is converting energy from one source to another.

    Again, please imagine a swimmer swimming either upstream or downstream with a current going "v". Same energy expenditure, but in one case he is going at speed 2v and in the other case 0.

    I'm expending energy sitting in my chair now. Am I therefore moving?
    Or does this mean if you sit at your desk your body will cool and you will die?

    In the train analogy, the excess heat comes from (1) the motor and (2) friction at different locations.

    The motor is not getting hot to push the earth around! It drives the wheels, converting more energy (via friction) into motion along the tracks.


    From the reference frame of the train, there is this rail and air zooming past whose friction against me would be accelerating me if I didn't expend energy to keep myself in the same place. Same as swimming upstream or running on a treadmill.

    Again, mount your rails on a table in an ice rink and you can get the train moving at any speed you like by sliding the table around at various speeds.

    ReplyDelete
  64. David, you're right about me. I am a physical chemist although my Ph.D. was in chemical physics. Be that as it may, I am not a trained physicist and claim no relativity expertise. Unfortunately, no one here appears to be able to claim expertise in relativity theory either (I wish some qualified physicist JBloggers such as Mike S. and Mechy Frankel would join the conversation). I also found your rejoinder to my wheel turning argument to be correct. However, the moving world scenario still suffers from unreality since in the real world, the train must have once started and must ultimately stop. In your alternative world scenario, that would mean that the world starts and stops coincident with the train. Besides that unreality, there is the fact that the world has many trains, some moving at different speeds and some not. How can the world accomodate all those differences simultaneously?

    ReplyDelete
  65. Hi David,

    You are showing extreme forbearance with some of us luddites.

    Again, please imagine a swimmer swimming either upstream or downstream with a current going "v". Same energy expenditure, but in one case he is going at speed 2v and in the other case 0.
    Nope, Different energy expenditure. Ke=1/2mv^2. If the velocity is different so is the Ke. Remember the distinction between vector analysis and absolute energy expenditure.
    I'm expending energy sitting in my chair now. Am I therefore moving?
    Yep. Heart is beating, diaphragm contraction, smooth muscle is contracting, axons are discharging. Not to mention all the chemical energy being released as we convert organic molecules into CO2 and H2O.
    Or does this mean if you sit at your desk your body will cool and you will die?
    If you do not do the above…Yep.
    From the reference frame of the train,…
    I will concede, as Y. Aharon has done, but only from that reference frame, and only if no acceleration EVER happens, and only in a frictionless system. i.e. an ideal not real system.
    Take your icetable as an example. The reason that you need it to be an ice table is because you need to reduce friction, so that no outside energy is required to overcome it.
    I was having a discussion with an engineer once. He posited this thought experiment. Imagine an alien viewing earth from space. He sees our cities and roads via the resolving power of his telescope. He will imagine, from looking at the traffic on our roads that the dominant beast on earth have a rigid exoskeletons and four wheels. The point is that for all practical purposes his conclusion will be correct and sufficient to explain his observations. That does not reflect reality however.

    Finally, as I keep saying, Occum's Razor is part of the epistemology of science

    ReplyDelete
  66. Y. Ben-David writes:
    I am aware that the modern atomic and quantum physics that was first developed more than a century ago is often defined as "physics that defies common sense" whereas the previously existing Newtonian physics did rely on common sense.

    Oh? Before Newton Aristotelian physics was the rule. Objects had natural and induced motions. So on. So forth. Inverse-square relations, the notion of the mathematical limit, Newtonian optics, the three laws of motion and so on were plenty radical in their day and defied "common sense".

    ReplyDelete
  67. I don't know what to say other than "no, there isn't". If you think this is so, please cite something.

    I fully conceded that (a) my choice of words was poor and (b) I am no expert on things physics.

    Nevertheless...

    We are arguing the distinction between an imaginary ideal world and the real world.

    In your imaginary world, the train does not accelerate nor does it experience friction. Energy is not expended to maintain the differential velocity. In the real world, this is not the case.

    As I said at the outset, the "time dilation" effect is experienced by the observer that accelerates. If two people stand adjacent to each other; one stands still and the other moves away, we can know with certainty who moved and who stayed still, since time will be different for the individual that accelerated away (slower).

    The outside observer can know with certainty who moved by measuring the the energy each individual consumed.

    I am going out on a limb here; The time dilation effect can be measured with atomic clocks. Place one clock at the top of the tower and the other clock in the basement. From the perspective of the person at the top of the tower they are not moving relative to the person at the bottom of the tower, yet we know from experiments that infact time passes slower at the top, i.e. the person at the top is moving faster than the person at the bottom. Although from the perspective of the person in the tower at the top they are not moving (relative to the ground) in absolute terms we know (can know) that they are.

    ReplyDelete
  68. See:
    http://home.fnal.gov/~pompos/light/light_page19.html

    ReplyDelete
  69. However, the moving world scenario still suffers from unreality since in the real world, the train must have once started and must ultimately stop. In your alternative world scenario, that would mean that the world starts and stops coincident with the train.

    I agree that the starting and stopping is the harder part to understand because it involves acceleration.

    However, the train doesn't in fact have to start and stop. Imagine the following scenario: You build a train track that circumnavigates the earth at the equator. You set the train in motion westward at a rate of one revolution per 23.9344696 hours (the sidereal day). Now the train is actually no longer rotating with respect to the fixed stars while the earth is. Who's moving now?

    Besides that unreality, there is the fact that the world has many trains, some moving at different speeds and some not. How can the world accomodate all those differences simultaneously?

    But the surprising fact is that it does. There are an infinite number of possible inertial frames and they all have different notions of length, time and simultaneity than the others. What remains fixed across all observers is the spacetime interval between two events x^2 + y^2 + z^2 - ct^2.

    ReplyDelete
  70. I haven't read every comment here, but here's my short, non-academic, response of the topic of Geocentrism.

    If I throw a rock at your head, and we end up in court and I tell the judge, "Your honor, the rock stood still, his head came at my rock really fast, and here's my notebook of calculations to prove it." Are you buying it?

    Gravity neatly explains the rotation of the planets around the sun. Other proposals, while mathematically feasible are lacking in elegance to explain the described motion.

    ReplyDelete
  71. how do geosynchronous satellites work in a geocentric world. is it mach principle

    ReplyDelete
  72. 'Even if Mach is right and even if a coordinate system with the earth unmoving can be worked out, Kepler was right and Ptolemy wrong a'

    maybe ptolemy was wrong but tycho brahe was right

    ReplyDelete
  73. Again, please imagine a swimmer swimming either upstream or downstream with a current going "v". Same energy expenditure, but in one case he is going at speed 2v and in the other case 0.

    Nope, Different energy expenditure. Ke=1/2mv^2. If the velocity is different so is the Ke. Remember the distinction between vector analysis and absolute energy expenditure.

    I'm afraid that you are still confused. It doesn't take energy to stay at the same velocity.

    I'm expending energy sitting in my chair now. Am I therefore moving?
    Yep. Heart is beating, diaphragm contraction, smooth muscle is contracting, axons are discharging. Not to mention all the chemical energy being released as we convert organic molecules into CO2 and H2O.

    And yet I'm not moving (meaning my center of mass is not moving). So the train doesn't have to be moving either in that sense. Anything above absolute 0 is moving in the respect that you mention.

    Or does this mean if you sit at your desk your body will cool and you will die?
    If you do not do the above…Yep.

    And yet I remain seated in position.

    From the reference frame of the train,…
    I will concede, as Y. Aharon has done, but only from that reference frame, and only if no acceleration EVER happens, and only in a frictionless system.
    i.e. an ideal not real system.


    I'm sorry that this is simply not true. Swimming upstream involves friction. Special relativity is not limited to frictionless environments.

    Take your icetable as an example. The reason that you need it to be an ice table is because you need to reduce friction, so that no outside energy is required to overcome it.

    No, I was just making a simpler example, if the table is on an electric cart, it still works.

    I was having a discussion with an engineer once. He posited this thought experiment. Imagine an alien viewing earth from space. He sees our cities and roads via the resolving power of his telescope. He will imagine, from looking at the traffic on our roads that the dominant beast on earth have a rigid exoskeletons and four wheels. The point is that for all practical purposes his conclusion will be correct and sufficient to explain his observations. That does not reflect reality however.

    Finally, as I keep saying, Occum's Razor is part of the epistemology of science


    This reasoning is not precise enough to conclude anything. Occam's Razer is a rule of thumb to help you make good guesses in the guess phase of the scientific method and can't be used to prove anything.

    ReplyDelete
  74. I don't know what to say other than "no, there isn't". If you think this is so, please cite something.

    I fully conceded that (a) my choice of words was poor and (b) I am no expert on things physics.

    Nothing to do with choice of words. Special relativity has no exceptions and all attempts to show it does are therefore bound to fail. If you want a book that teaches you how to navigate the paradoxes of special relativity with high school math, I suggest "Spacetime Physics". I think that further discussion here along these lines is not going to move us further towards the truth...

    ReplyDelete
  75. David, While your imaginative constructions are interesting and may lead to a better understanding of relativity, they still do not reflect the real world. A train, no matter how well supplied with fuel, must start sometime and will come to a halt at some later time. The stated condition of constant velocity can't be accomplished independent of time. That putative train must achieve its stated velocity corresponding to 1000 mph from start in some time interval. For that time interval, the passengers in the train would feel acceleration forces and would note a change in the relative motion of the ground. Observers on the ground, however would not feel those acceleration forces. They should correctly conclude that the train is moving rather than they (relative to the earth. The same conclusion would be reached by the train passengers.

    ReplyDelete
  76. David, While your imaginative constructions are interesting and may lead to a better understanding of relativity, they still do not reflect the real world. A train, no matter how well supplied with fuel, must start sometime and will come to a halt at some later time. The stated condition of constant velocity can't be accomplished independent of time. That putative train must achieve its stated velocity corresponding to 1000 mph from start in some time interval. For that time interval, the passengers in the train would feel acceleration forces and would note a change in the relative motion of the ground. Observers on the ground, however would not feel those acceleration forces. They should correctly conclude that the train is moving rather than they (relative to the earth. The same conclusion would be reached by the train passengers.

    Let's assume, arguendo, that you are correct. Then the train and the stars are rotating around the earth with a period of one sidereal day, while the earth is stationary. That is ... Geocentrism!

    ReplyDelete
  77. nachman said...
    how do geosynchronous satellites work in a geocentric world. is it mach principle


    The rotation of stars causes both the earth to bulge at the equator and the goe-stationary satellites to hang there.


    'Even if Mach is right and even if a coordinate system with the earth unmoving can be worked out, Kepler was right and Ptolemy wrong a'


    maybe ptolemy was wrong but tycho brahe was right

    Brahe was also wrong. Kepler figured out the ellipse thing after Brahe died. You could have a "neo-Tychonic" system which is the usual system but the restated from the PoV of the earth.

    ReplyDelete
  78. I haven't read every comment here, but here's my short, non-academic, response of the topic of Geocentrism.

    If I throw a rock at your head, and we end up in court and I tell the judge, "Your honor, the rock stood still, his head came at my rock really fast, and here's my notebook of calculations to prove it." Are you buying it?


    I think that is beside the point. If you shove a large boulder onto a roadway in advance of oncoming car, you are still responsible.

    ReplyDelete
  79. While your imaginative constructions are interesting and may lead to a better understanding of relativity, they still do not reflect the real world. A train, no matter how well supplied with fuel, must start sometime and will come to a halt at some later time. The stated condition of constant velocity can't be accomplished independent of time.

    This is also true of someone trying to remain stationary in a raging river. Or standing still in a hurricane on roller stakes.

    Also it depends on how cleverly you build your train. A Vactrain might be feasible and you could power it with solar energy.

    Anyhow the point is that, yes, it is hard to remain stationary when there is a 5.97219E24 kilogram mass rotating under you. That does't prove that you are moving.

    ReplyDelete
  80. David, this is getting a bit repetitive and tiresome. I only mentioned that the ground observer of your moving train is stationary relative to the earth - not the stars. He and earth are spinning in space at 1000 mph and moving about the sun and galactic center at much higher speeds. My point, and you have conceded as much, is that in real life, this model train of yours would have started at some point and accelerated to your putative 1000 mph in an east to west direction. During that acceleration, passengers on the train would have felt themselves pushed back due to an inertial force. They could then conclude that they must be accelerating. The observers on the ground would not feel such a force. Hence, they could conclude that they were standing still relative to the earth. The conclusion that both groups would reach is that the train group was moving relative to the ground group. This should be true even in general relativity.

    As to the issue of geocentrism, we have already discussed the reasons for not considering that the entire vast universe rotates about the earth. It doesn't matter if one could arrive at a scheme wherein such a situation was defensible. It is still an unrealistic and complicated defense that produces no new knowledge. Einstein's insistence that the laws of physics must be formulated independent of frame of reference did lead to new advances in physics, but does not, by itself, justify the alternative universe scenario.

    ReplyDelete
  81. Y.Aharon,

    David clearly believes in perpetual motion machines.

    ReplyDelete
  82. Y. Aharon and Yossi,

    It is helpful to make specific coherent arguments rather than making general assertions what is helpful and not in forming models of the universe. You are both arguing that our common sense notions of who is moving and who remains stationary have some form of validity, but unfortunately, experiment says otherwise. I agree that it is somewhat tiresome to continue to show why your examples have problems because we know from the relativity that there must be such a hole in your argument; otherwise, as I've mentioned, a Nobel prize awaits.

    To summarize your arguments (actually, to be blunt, fallacies)

    1) You can tell which of two inertial frames is the moving one because the actors in one frame are expending energy (or more energy). Since a moving train is burning fuel, it is must be the one that is moving while the earth is stationary.

    As I've mentioned many times, this is very easy to disprove: a swimmer swimming upstream has to expend energy to remain stationary relative to the ground.

    2) The actor who will eventually have to give up due to friction is the one who is moving. Since the train eventually runs out of fuel, it must be moving.

    The same examples suffice, since this a variation on the same fallacy.

    3) The actor who was previously accelerated must be moving. Since the train started from a standstill relative to the ground and must accelerate, it must be moving.

    Generally speaking, to move from one inertial from to any other requires acceleration; therefore no single frame is preferred. Going from a moving state to a non-moving state relative to any frame requires acceleration (aka stopping), so by this criteria, no frame is 'at rest'. Also, since we've all accelerated many times, this would mean that none of us are stationary. Also the earth is decelerating, so therefore it must be moving as well. Undoubtedly, the earth was accelerated more violently when the collision which created the Moon occurred.

    Finally, if acceleration is your criteria, then the earth is the one that is moving and not the 'sidereal period' train, since the earth is rotating and thus accelerating centripetally, while the train is not.

    4) If you can look at either of two inertial frames as stationary, then you could pick an infinite number of different frames as stationary. There are many trains moving in different directions.

    The answer is that yes, there are an infinite number of equally valid frames, each of which will have different views of, for example, simultaneity, length, and time measurements. Unfamiliar, but absolutely true.

    5) Relativity violates the laws of thermodynamics, or has an exception for frames with friction, or adherence to relativity requires belief in perpetual motion machines.

    This is just a fallacy about relativity.

    6) But we know that the train is moving relative to the Earth.

    Yes, we know that train is moving relative to the earth. We also know that the earth is moving relative to the train.

    So the bottom line is that our intuition as to "who is moving" can't help us here. What can perhaps help us is acceleration and "fictional forces" implied by rotation. That is where general relativity steps on and I step off.

    "I have two things to say that might surprise you: first, geocentrism is a valid frame of reference, and second, heliocentrism is not any more or less correct." Right or wrong, that is not coming from me.

    ReplyDelete
  83. David,

    I think you are incorrect on a number of points, but I do not have time to refute you point by point. I merely refer you again to Dr. Carlop's own words below:
    -----------------

    There are at least two distinct issues: can we detect "absolute" rotation with local measurements, and can we detect rotation if we are allowed to look at the distant Universe?

    For the first question, my answer is, quite firmly, "I don't know." Local experiments such as the Foucault pendulum or Gravity Probe B, see effects that are most easily attributed to rotation. But it's an open question whether general relativistic effects of distant rotating matter could reproduce the same results.

    There are hints in both directions. For instance, in the solution of the Einstein field equations for an isolated spherical mass in an otherwise empty universe, we can unambiguously tell whether the mass is rotating. This can't be an effect of other matter, because in this solution there is no other matter. On the other hand, if we take this same isolated spherical mass – say, nonrotating – and put it inside a massive rotating shell, in an otherwise empty universe, we will see effects such as a Coriolis force that are "induced" from the rotating shell and that mimic rotation of the mass.

    At the very least, the answer depends on the setting (in particular, on boundary conditions). It remains possible that general relativity is "Machian" – unable to locally identify absolute rotation – if the Universe is spatially closed. As I said in my previous post, it's not entirely clear even how to pose the question.

    For the second question – can we detect rotation if we are allowed to look at the distant Universe? – the answer is certainly "yes," as long as we are allowed to assume causality. In particular, changes in local rotation have identifiable local causes, and these can be attributed to the distant Universe only if we allow these local causes to act backwards in time (or if you reject causality altogether).
    ------------
    Clearly, you do not believe in "time's arrow," or the notion of causality. You may argue that you do, but your arguments prove that you do not. The ability to write correct and solvable equations does not imply that those equations model physical laws in the real world. Some solutions are simply wrong, even if they may seem to be mathematically correct - unless we are all to believe like you do, that there is no absolute notion of causality. Most intelligent people are simply unable to accept this, regardless of relativistic arguments that produce solvable equations. Perhaps you know something about causality that the rest of us are missing.

    So yes, a geocentric world can be made mathematically correct, and may even be a reasonable fit, depending on the problem you are studying - e.g., when you assume that there is only a handful of simple objects in the universe. However, it is a terrible frame of reference for anything occurring off the surface of the earth. And it certainly is not a good approximation for calculations involving the known universe. It leads to absurd conclusion about cause and effect that can not be generalized into provable physical laws.

    ReplyDelete
  84. David, you write as if you were an expert on relativity theory, whereas none of us are - as far as I can see (your equation for a spacetime interval, for example, has an incorrect time term and should really be written in differential form). You also persist in dealing with specific parts of a scenario, rather than the larger picture. A train must have started some time and must therefore have accelerated to the presumed westerly constant speed of 1000 mph. During that acceleration anyone on the train would be pushed back due to a strong inertial force. Anyone on the ground in the train's vicinity during that acceleration would not feel such a push. Hence both groups would conclude that the train was moving relative to the ground rather than the ground relative to the train. The ground observers would, however, note that the sun or stars appear to be moving in a westerly direction. The observers on the train moving west at 1000 mph would see a stationary heaven. The difference in perspective is due to their different reference frames. Relativity theory, as I understand it, would claim that the physics of the relative motions of the earth, sun, and stars must be formulated in a way that is independent of reference frame - not that the train can't be said to be moving relative to the ground or that geocentrism is a ralistic model of nature. Realism is important in science, even if there are phenomena that are counter intuitive. Einstein, himself, attacked some modern quantum concepts as being physically unrealistic. The notion that the universe rotates around a stationary earth is the height of such unrealism.

    ReplyDelete
  85. AL said...
    David,

    I think you are incorrect on a number of points, but I do not have time to refute you point by point. I merely refer you again to Dr. Carlop's own words below:


    AL, you misunderstand me. I don't understand General Relativity, so I don't take a position. I only quote others. This quotation is not from me: "I have two things to say that might surprise you: first, geocentrism is a valid frame of reference, and second, heliocentrism is not any more or less correct."

    In this thread, my only argument is against the commenters who are making simplistic and fallacious arguments that imply a preference for on or another inertial frames. These are simply wrong. If the train is going at constant velocity, then there is no preference. And, ironically, if there was for a moving train and fixed earth, then that would be an argument for Geocentrism.

    ReplyDelete
  86. David, you write as if you were an expert on relativity theory, whereas none of us are - as far as I can see (your equation for a spacetime interval, for example, has an incorrect time term and should really be written in differential form).

    I'm not an expert, but the arguments here are fallacious even with Galilean relativity.

    Yes I left off ^2 in my transcription. You will find the equation in coordinate forms, e.g. 17-3 on this page http://www.feynmanlectures.caltech.edu/I_17.html.

    You don't need to be an expert to understand that there is no way to prefer one or another inertial frame. Again, I picked out every specific argument made and showed an equivalent example going the other way (as relativity predicts there must be).

    You also persist in dealing with specific parts of a scenario, rather than the larger picture. A train must have started some time and must therefore have accelerated to the presumed westerly constant speed of 1000 mph. During that acceleration anyone on the train would be pushed back due to a strong inertial force. Anyone on the ground in the train's vicinity during that acceleration would not feel such a push. Hence both groups would conclude that the train was moving relative to the ground rather than the ground relative to the train.

    I addressed that explicitly. The accelerating train is *not* an inertial frame. The question is once it reaches a constant velocity. There is no preference to say whether it accelerated from 0 to 1000 or decelerated from 1000 to 0 (in a straight line).

    The ground observers would, however, note that the sun or stars appear to be moving in a westerly direction. The observers on the train moving west at 1000 mph would see a stationary heaven. The difference in perspective is due to their different reference frames. Relativity theory, as I understand it, would claim that the physics of the relative motions of the earth, sun, and stars must be formulated in a way that is independent of reference frame - not that the train can't be said to be moving relative to the ground or that geocentrism is a ralistic model of nature.

    Of course the train is moving relative to the ground, just as the ground is moving relative to the train. And they are both moving in other reference frames. But with constant velocity motion, there is no preference for one frame or the other. If "realism" tells you otherwise, then "realism" is misleading you.

    When you factor in the rotation of the earth, and if there is a true preference for the Heliocentric frame, then the train should be considered stationary with the earth moving under it, since is not accelerating, while the rotating earth appears to be.

    Realism is important in science, even if there are phenomena that are counter intuitive. Einstein, himself, attacked some modern quantum concepts as being physically unrealistic. The notion that the universe rotates around a stationary earth is the height of such unrealism.

    The problem with Geocentrism has nothing to do with realism. It has to do with the explaining the "fictitious" forces in non-inertial frame (Newton's bucket). Again, to repeat, I don't understand General Relativity, but some physicist writers seem to indicate that this can be explained too (I'm making no such claim).

    Your Einstein analogy is apt, because as best an anyone can tell, Einstein was wrong, and quantum theory correct.

    ReplyDelete
  87. David, you admit to not understanding General Relativity, yet you persist with your constant speed train analogy as a model for the issue of geo vs. heliocentrism. The latter is an issue for General Relativity since the (vector) motions involved are not uniform.
    When you are confronted with the matter of the train requiring acceleration from start to reach its extremely high velocity (1000 mph), you obfuscate. Here are some of your citations:
    "Generally speaking, to move from one inertial from to any other requires acceleration; therefore no single frame is preferred. Going from a moving state to a non-moving state relative to any frame requires acceleration (aka stopping), so by this criteria, no frame is 'at rest'. Also, since we've all accelerated many times, this would mean that none of us are stationary. Also the earth is decelerating, so therefore it must be moving as well. Undoubtedly, the earth was accelerated more violently when the collision which created the Moon occurred." - David

    "I addressed that explicitly. The accelerating train is *not* an inertial frame. The question is once it reaches a constant velocity. There is no preference to say whether it accelerated from 0 to 1000 or decelerated from 1000 to 0 (in a straight line)." - David

    What do those citations even mean, and how do they counter the argument that in an accelerating train, the passengers can feel a force that would inform them of their state of motion and how it has changed from the train's start time?

    Your assertion that objects in relative uniform motion will find that the same laws of physics apply to them is but a statement of the Special Relativity theory. I don't contest that. The issue of a stationary earth with a rotating vast universe vs. a spinning earth is, however, not reducible to such relative uniform motion depictions.

    Finally, your mantra of relativity theory overriding any conventional ideas of realism was countered by my Einstein citation vis-a-vis some ideas underlying the 'new' quantum theory, i.e., as charged with being magical and unrealistic. In other words, Einstein believed his theories to reflect reality, despite some unconventional aspects. As you state, the mathematics of quantum mechanics appears to successfully predict experimental results despite some counter-intuitive or metaphysical (multi-universe) interpretations. That just vindicates the mathematics, not the physical model. The same could probably be said about a Machean view of the universe - if one could actually develop the mathematics that it implies.

    ReplyDelete
  88. David, you admit to not understanding General Relativity, yet you persist with your constant speed train analogy as a model for the issue of geo vs. heliocentrism.

    Not at all. I have said very clearly at least 5 times that accelerating frames (and rotating frames are accelerating centripetally) are confusing to me because some physicists (who I quoted) seem to imply that they are clearly accelerating, while others seem to say that you can ascribe the fictitious forces to gravity and it all works out. See my other post for the quotes. I simply don't understand it well enough or why it is that the quotations from people who know better don't align.

    The latter is an issue for General Relativity since the (vector) motions involved are not uniform.

    Yes, they are accelerating so there are fictitious forces as I've said many times. The vector has nothing to do with it. Velocity is a vector, but frames with a constant velocity cannot be labeled as being in absolute motion as accelerating frames can (maybe).

    When you are confronted with the matter of the train requiring acceleration from start to reach its extremely high velocity (1000 mph), you obfuscate.

    I'm not obfuscating. While the train is accelerating, it is in an accelerating frame. Once it reaches its constant velocity, it is in an inertial frame like any other. There is no "memory" of what came before (this is actually a postulate of special relativity). Therefore, the prior acceleration cannot tell you that the train is "the one that is moving".


    Here are some of your citations:
    "Generally speaking, to move from one inertial from to any other requires acceleration; therefore no single frame is preferred.


    Therefore the prior acceleration is irrelevant.

    Going from a moving state to a non-moving state relative to any frame requires acceleration (aka stopping), so by this criteria, no frame is 'at rest'.

    Therefore the prior acceleration is irrelevant.

    Also, since we've all accelerated many times, this would mean that none of us are stationary.

    Therefore the prior acceleration is irrelevant.

    Also the earth is decelerating, so therefore it must be moving as well.

    Therefore the prior acceleration is irrelevant.

    Undoubtedly, the earth was accelerated more violently when the collision which created the Moon occurred." - David

    Therefore the prior acceleration is irrelevant.

    "I addressed that explicitly. The accelerating train is *not* an inertial frame. The question is once it reaches a constant velocity. There is no preference to say whether it accelerated from 0 to 1000 or decelerated from 1000 to 0 (in a straight line)." - David

    Therefore once the train reaches "sidereal speed", the prior acceleration is irrelevant.

    What do those citations even mean, and how do they counter the argument that in an accelerating train, the passengers can feel a force that would inform them of their state of motion and how it has changed from the train's start time?

    They don't, and I wasn't implying that. I was countering the argument that a constant velocity train must be the one that is moving because it had accelerated in the past (or because it will run out of fuel and be accelerated by the rotating earth under it in the future).

    Your assertion that objects in relative uniform motion will find that the same laws of physics apply to them is but a statement of the Special Relativity theory. I don't contest that. The issue of a stationary earth with a rotating vast universe vs. a spinning earth is, however, not reducible to such relative uniform motion depictions.

    I agree and I never said it was; the reason is because the Earth is rotating and thus accelerating. I was countering the arguments proffered above, about heat, friction, running out of fuel, realism, infinite number trains, etc, which apply equally to the train in an inertial frame and therefore must be fallacious.

    ReplyDelete
  89. Finally, your mantra of relativity theory overriding any conventional ideas of realism was countered by my Einstein citation vis-a-vis some ideas underlying the 'new' quantum theory, i.e., as charged with being magical and unrealistic. In other words, Einstein believed his theories to reflect reality, despite some unconventional aspects. As you state, the mathematics of quantum mechanics appears to successfully predict experimental results despite some counter-intuitive or metaphysical (multi-universe) interpretations. That just vindicates the mathematics, not the physical model. The same could probably be said about a Machean view of the universe - if one could actually develop the mathematics that it implies.

    Einstein thought that quantum theory was actually wrong, but it is actually right, and he was wrong, as best we can tell. The EPR experiment really does give the results that Einstein thought would be impossible. God does actually appear to throw dice.

    The problem with the Machean view is that it is not a precise theory and when made precise, it may be wrong (or perhaps it is definitely wrong).

    If you tell me that geocentrism seems wrong because the earth bulges at the equator, then I can't dispute you, although there seem to be some physicists that do (I gave the quotations).

    If you tell me that geocentrism is wrong because relativity is a formalism and, in "reality", you can tell that a train in an inertial frame is the one that is moving, then you are mistaken.

    That's all I was saying :).

    ReplyDelete
  90. David, for the hopefully final time. Einstein proposed some thought experiments which involved what the observer in a certain situation would perceive. He used those gedanken 'results' as the basis for reformulating physical theory. Now, you tell me that what the observer felt before a constant state of motion was obtained is irrelevant. Of course its relevant and it was relevant to Einstein. He was just focused on a different point about relative uniform motions. You can't dissociate memory from the perception of reality. In the end, we don't disagree about whether or not the earth is stationary in space. It's just a question of how to demonstrate the earth's motion.

    ReplyDelete
  91. if 2 people drop an apple simultaneously from the north and south pole and one says the north pole moved towards the apple. what happened with the south pole.

    ReplyDelete
  92. does aberration of starlight disprove geocentrism.?

    if you put focaults' pendulum on different planets and it made one revolution per one rotation of the planet. whether one day or 30 days. would that disprove geocentrism ?

    ReplyDelete
  93. does aberration of starlight disprove geocentrism.?

    if you put focaults' pendulum on different planets and it made one revolution per one rotation of the planet. whether one day or 30 days. would that disprove geocentrism ?


    It would seem to at first, as would the Earth's bulge at the equator. However, see the referenced blog post which casts doubt on this. This is above my pay grade.

    ReplyDelete
  94. noson said...
    if 2 people drop an apple simultaneously from the north and south pole and one says the north pole moved towards the apple. what happened with the south pole.


    It also moved.

    As an aside, in a Newtonian world, the earth and apple would actually fall towards each other in order to conserve momentum. It's just that the earth's greater mass would mean it is moving much more slowly.

    ReplyDelete
  95. David, for the hopefully final time. Einstein proposed some thought experiments which involved what the observer in a certain situation would perceive. He used those gedanken 'results' as the basis for reformulating physical theory.

    The thought experiment is a historical artifact and an intuitive way to derive the results. Actual measurements show that the results are true in "real life".

    Now, you tell me that what the observer felt before a constant state of motion was obtained is irrelevant.

    If not, then I would know that I am moving as I am sitting in my chair, since I had to accelerate my car to get it to stop in front of my house. I as pointed out, the the violent collision that spawned the moon from the earth surely caused a noticeable acceleration to the earth itself. If "memory" is important, then it should be just as relevant in those examples.

    Of course its relevant and it was relevant to Einstein. He was just focused on a different point about relative uniform motions. You can't dissociate memory from the perception of reality.

    One of the postulates of special relativity is "memorylessness". I'm trying to track down the source where that was defined better; I can now only find the wikipedia page reference.

    Anyhow, I agree that we "perceive" the earth as stationary. This was one of the reasons why Geocentrism was so hard to overcome. I'm not sure why you insist on considering very important a psychological tendency which actually retarded acceptance of Heliocentrism.

    In the end, we don't disagree about whether or not the earth is stationary in space. It's just a question of how to demonstrate the earth's motion.

    There is no question that if one want to demonstrate the Earth's motion in modern times, one will refer to the acceleration involved in both the the rotation on its axis and the revolution around the Sun (or the center of mass of the Sun-Earth system).

    The issue remains that even this acceleration can be viewed as gravity, and then I get lost among the seemingly contradictory popular explanations of GR.

    ReplyDelete
  96. does aberration of starlight disprove geocentrism.?

    Actually, to be more precise, the aberration of starlight proves movement with respect to the "fixed" stars. Changes in aberration over the course of the year proves acceleration wrt the fixed stars over time.

    ReplyDelete
  97. I did not quite understand your answer so I will rephrase. if 2 people drop an apple simultaneously from the north and south pole and one takes the view that the north pole (earth)moved north towards 'its' apple then by definition for this to happen the south pole (earth) will have also to move north simultaneously fleeing 'its' apple so the south pole apple should not land on the ground.

    does the fact that the south pole apple lands on the ground means we can determine in this case absolute movement ?

    ReplyDelete
  98. if aberration of starlight proves movement with respect to the "fixed" stars

    then is this not against geocentrism which insists on a fixed earth

    ReplyDelete
  99. Anonymous noson said...
    if aberration of starlight proves movement with respect to the "fixed" stars

    then is this not against geocentrism which insists on a fixed earth


    Not if you view the earth as stationary and the stars as moving.

    ReplyDelete
  100. noson said...
    I did not quite understand your answer so I will rephrase. if 2 people drop an apple simultaneously from the north and south pole and one takes the view that the north pole (earth)moved north towards 'its' apple then by definition for this to happen the south pole (earth) will have also to move north simultaneously fleeing 'its' apple so the south pole apple should not land on the ground.

    does the fact that the south pole apple lands on the ground means we can determine in this case absolute movement ?


    No, it just means that in the north apple frame, the south apple is accelerating at 2g. In the north apple frame, there is a general "fictitious" northward acceleration of g. This is the part that might prove "absolute" acceleration.

    ReplyDelete
  101. if in the north apple frame, the south apple is accelerating at 2g then it is breaking newton's law of gravity. in other words geoncentricty has been falsified

    ReplyDelete
  102. when you say aberration of starlight proves movement with respect to the "fixed" stars.

    I thought you meant movement of earth
    with respect to the "fixed" stars.

    or did you just mean there is relative movement between the two

    ReplyDelete
  103. Noson, you are right in an accelerating frame, you will see an extra "fictitious" force that cause the other apple to fall at 2g. It may very well be this is enough to prove you are not at rest. I refer you to the references that I give in the other post for you to decide that for yourself. I am out of my depth there.

    ReplyDelete
  104. I do not understand how in a fixed earth tides can change the length of a day

    http://bowie.gsfc.nasa.gov/ggfc/tides/intro.html

    Tides affect the earth's rotation in two sharply contrasting ways. One way, caused by tidal friction, produces an extremely slow secular change in rotation. The other way, caused by the continual movements of the tides about the planet, produces very small but very rapid changes in rotation. These rapid changes occur at exactly the same periods as the tides themselves -- half-daily, daily, etc. (The IERS Special Bureau for Tides is concerned primarily with the rapid changes, but some of our data have implications for the secular changes.)

    ReplyDelete

Comments for this blog are moderated. Please see this post about the comments policy for details. ANONYMOUS COMMENTS WILL NOT BE POSTED - please use either your real name or a pseudonym.

Have you not been receiving my latest posts?

This is for those who receive my posts via email and have not seen posts in the last few days. The reason is because I moved over to a new s...